Greatest common factor of 35 and 39

Answers

Answer 1

Answer:

The GCF of 35 and 39 is 1.

Step-by-step explanation:

Factors of 35: 1, 5 and 7

Factors of 39: 1, 3, 13


1 is the only factor that these numbers have in common.

Hope this helps!

Answer 2

Answer:

35 - 1, 5, 7

39 - 1, 3, 13

the greatest common factor will be 1

hope this helps


Step-by-step explanation:



Related Questions

Mrs.Davis bought 6 pounds of grapes for 18 dallors at this rate how much would 72 ounces of grapes have cost

Answers

Answer:

Cost of 72 ounces of grapes is $13.5


Step-by-step explanation:

In order to solve his problem, we first need to get all the values in the same units.

  1 pound = 16 ounces

∴ 6 pounds= 16×6 = 96 ounces

Now by unitary method, we solve the question as follows:

price of 96 ounces of grapes = $ 18

price of 1 ounce of grapes = $ (18÷96)

∴ price of 72 ounces of grapes = $ [(18÷96)×72]

= (0.1875×72) =  $ 13.5

find the sum of the geometric series. 14-7+(7/2)-(7/4)+

Answers

Answer:

28/3

Explanation:

Common ratio of the series is −12. Using formula for sum of an infinite geometric convergent series a1−r, the required sum would be 141+12=283

p-2 times what = 4-p^2

Answers

Answer:

-2-p

Step-by-step explanation:

(p-2)x=4-p^2

(4-p^2)/(p-2)

-2-p

The solution of the expression for the given condition would be, - (p + 2).

Used the concept of trigonometry formula which stated that,

[tex](a - b)^2 = a^2 - 2ab + b^2[/tex]

Let us assume that,

When we multiply by x in (p - 2) the solution is (4 - p²).

So, it can be written in mathematical form as,

[tex]x \times (p - 2) = (4 - p^2)\\[/tex]

[tex]x \times (p - 2) = (2^2 - p^2)\\[/tex]

[tex]x \times (p - 2) = (2 - p) (2 + p)[/tex]

[tex]x \times (p - 2) = - (p - 2) (2 + p)[/tex]

Divide both sides by (p - 2),

[tex]x = - (p + 2)[/tex]

So, we get;

[tex](p - 2) \times - (p + 2) = (4 - p^2)[/tex]

To learn more about the divide visit:

https://brainly.com/question/28119824

#SPJ6


Ryan rides his bike for 30 minutes and travels a distance of 4 miles. How fast was Joe traveling?

Answers

Answer: 8 miles per hour


Step-by-step explanation:

30 + 30 = 60, 60 minuets in an hour  4 + 4 = 8. 8 miles per hour hope i helped :)

Answer:

8 miles!

Step-by-step explanation:

30 + 30 = 60, 60 minutes in an hour and  4 + 4 = 8. So the answer is 8 miles per hour


Hope this helps!

In triangle LNP, R is the centroid and LO = 30. Find LR and RO. Enter the answers as numbers.

Answers

Answer:

[tex]|LR|=20[/tex]

[tex]|RO|=10[/tex]

Step-by-step explanation:

The centroid divides the median in the ratio [tex]2:1[/tex].


Thus [tex]|LR|:|RO|=2:1[/tex]

This implies that,

[tex]|LO|:|RO|=3:1[/tex]

We were given that,

[tex]|LO|=30[/tex]

We substitute this value to obtain,

[tex]30:|RO|=3:1[/tex]


or


[tex]\frac{30}{|RO|}=\frac{3}{1}[/tex]

We cross multiply to get,

[tex]3|RO|=30\times 1[/tex]


We divide through by 3, to get,


[tex]|RO|=10[/tex]



We can observe from the diagram that

[tex]|LR|+|RO|=|LO|[/tex]


[tex]|LR|+10=30[/tex]


[tex]|LR|=30-10[/tex]


[tex]|LR|=20[/tex]




PLS HURRY GOT 10 MINUTES! JUST NEED THESE TWO QUESTIONS AND I DON'T KNOW THEM! WILL GIVE BRAINLIEST!!


1. What is the linear function equation represented by the graph?
f(x)=

2. classify each sequence as arithmetic, geometric, or neither by dragging it into the correct box.

Arithmetic Geometric Neither

−4,−2,1,4,8,… −1,3,7,11,15,… 36,12,4,4/3,4/9,…

Answers

Answer:

1) f(x) = -1/2x+1

I'm sorry I don't know the 2nd one...

Step-by-step explanation:

The y-intercept is always plotted first. from there, you go down one, and to the right too. and so one, and in reverse, up one, and left two.

Answer:1. f(x) = -1/2x+1




Find X
I have no idea how to make it

Answers

Answer: 5

Step-by-step explanation:

The length of the midsegment is one-half the length of the base.  So,

Base = 2(midsegment)

7x + 1 = 2(4x - 2)

7x + 1 = 8x - 4

       1 =  x - 4

       5 = x


Do 7 plus 1 then do 4 time 2 then subtract the two answers and u have 0 which is x

If matrix A has dimensions m x n and matrix B has dimensions n x p where m, n, and p are distinct positive integers, which of the following is true?
I.the product of BA doesn't exist
II. the product of AB exists and has dimensions m x p
III. the product of AB exists and has dimensions n x n

a) I only
b) II only
c) III only
d) I and II only
e) I and III

Answers

You probably already have an idea of what a matrix is; it's a rectangular array of numbers. What they represent is a bit complicated to explain. There's a whole subject about it (see "linear algebra" for more info). Whatever they represent isn't important though, you don't need to know everything about matrices to compute their product (or whether it's even possible).

A quick definition: A matrix of dimension [tex]r[/tex]-by-[tex]c[/tex] is a matrix with [tex]r[/tex] rows and [tex]c[/tex] columns.

Matrix multiplication all comes down to an operation called the "dot product". It's defined by the sum of component-wise products of elements between two lists. What this means is, if [tex]x=\{1,2,0\}[/tex] and [tex]y=\{-1,0,3\}[/tex], then the dot product of [tex]x[/tex] and [tex]y[/tex] is

[tex]x\cdot y=(1)(-1)+(2)(0)+(0)(3)=-1+0+0=-1[/tex]

What we did was take the first elements of each list and multiplied them, and the same for the other two elements, then added them all together. Notice that the product can't be computed if [tex]x[/tex] and [tex]y[/tex] don't thave the same number of elements.

We write this product in matrix form as

[tex]\begin{bmatrix}1&2&0\end{bmatrix}\begin{bmatrix}-1\\0\\3\end{bmatrix}[/tex]

Notice the pattern here: on the left, a matrix with 1 row and 3 columns; on the right, a matrix with 3 rows and 1 column. The number of columns of the first matrix have to match the number of rows of the second.

The orientation makes a big difference. The product above returns a 1-by-1 matrix (or simply a scalar number):

[tex]\begin{bmatrix}1&2&0\end{bmatrix}\begin{bmatrix}-1\\0\\3\end{bmatrix}=\begin{bmatrix}(1)(-1)+(2)(0)+(0)(3)\end{bmatrix}=\begin{bmatrix}-1\end{bmatrix}=-1[/tex]

On the other hand, the alternate orientation would result in a 3-by-3 matrix.

[tex]\begin{bmatrix}1\\2\\0\end{bmatrix}\begin{bmatrix}-1&0&3\end{bmatrix}=\begin{bmatrix}(1)(-1)&(1)(0)&(1)(3)\\(2)(-1)&(2)(0)&(2)(3)\\(0)(-1)&(0)(0)&(0)(3)\end{bmatrix}=\begin{bmatrix}-1&0&3\\\-2&0&3\\0&0&0\end{bmatrix}[/tex]

So the number of rows of the first matrix and number of columns of the second matrix determine the number of rows and columns, respectively, of the matrix product.

I think we have enough information about matrix multiplication to answer this question. If [tex]A[/tex] has dimensions [tex]m\times n[/tex] and [tex]B[/tex] has dimensions [tex]n\times p[/tex], then the matrix product [tex]AB[/tex] exists ([tex]n[/tex] columns in [tex]A[/tex], [tex]n[/tex] rows in [tex]B[/tex]), but the matrix product [tex]BA[/tex] does not ([tex]p[/tex] columns in [tex]B[/tex], [tex]m[/tex] rows in [tex]A[/tex], but [tex]m\neq p[/tex]). So I is not true.

We know [tex]AB[/tex] exists, and with [tex]m[/tex] rows in [tex]A[/tex] and [tex] p[/tex] columns in [tex]B[/tex], we expect [tex]AB[/tex] to have [tex]m[/tex] rows and [tex]p[/tex] columns, so [tex]AB[/tex] has dimensions [tex]m\times p[/tex]. So II is true.

With dimensions [tex]r\times c[/tex], a matrix would contain [tex]rc[/tex] elements. [tex]m,n,p[/tex] are distinct, so [tex]mp\neq n^2[/tex]. So III is not true.

Answer:  The correct option is

(d) I and II only.

Step-by-step explanation:  Given that matrix A has dimensions m x n and matrix B has dimensions n x p where m, n, and p are distinct positive integers.

We are to select the one that is true from the following :

I.   the product of BA doesn't exist

II.  the product of AB exists and has dimensions m x p

III. the product of AB exists and has dimensions n x n

We know that two matrices X and Y can be multiplied if the number of columns in X is equal to the number of rows in Y.

Also, if X has dimensions  a x b and Y has dimensions b x c, then the product XY is possible and it has dimensions a x c. Also, the product YX doesn't exist.

So, for the given matrices A and B, the following points are true :

(I) the product of BA doesn't exist.

(II) the product of AB exists and has dimensions m x p.

Thus, only I and II are TRUE.

Option (d) is CORRECT.

Justin's rice ball recipe uses 100 grams of rice to make 11 rice ball. Justin has 700grams of rice. How many rice balls can Justin make with 700 grams of rice?

Answers

Answer: I TINK 77 rice balls


Step-by-step explanation:


Answer:77

Step-by-step explanation:

Can someone help me with these please?

Answers

Point-slope form: y-y1 = m(x-x1)

Standard form: ax + by = c

Slope-intercept form: y = mx+b


Start by finding the slope. We know it is negative since the line is decreasing. The slope is -4/3.


To create point-slope form, we need to get one point from the graph. Let's use (3,0).

[tex]y = -\frac{4}{3}(x-3)[/tex]


To create slope-intercept form, we need the slope and the y-intercept. The y-intercept is the point where our equation crosses the y-axis. For this equation, it is 4.

[tex]y = -\frac{4}{3}x + 4[/tex]


To get standard form, solve the equation in terms of C.


Point-slope form: y = -4/3(x-3)

Slope-intercept form: y = -4/3x + 4

Standard form: 4/3x + y = 4



What is the distance between the two points (5,-2) and (-3,8)

Answers

you need to use the distance formula
[tex]d = \sqrt{ {(x - x)}^{2} + {(y - y)}^{2} } [/tex]

[tex] \sqrt{ {(5 + 3)}^{2} + {( - 2 - 8)}^{2} } [/tex]
so the distance between points (5,-2) and (-3,8) is
[tex]2 \sqrt{41} [/tex]
which won't simplify so it stays as is

10 cookies and 3 brownies cost $11.25 6 cookies and 9 brownies cost $15.75 what is the cost of one brownie?

Answers

Greetings!

Answer:

One brownie costs $1.25

Step-by-step explanation:

We need to solve this with simultaneous equations. First, write the equations out in two lines:

c = cookies and b = brownies.

10c + 3b = 11.25

6c + 9b = 15.75

We need to make either the two c values the same or the two b values the same. Lets change the two b values to the same by multiplying the first equation by three:

10c + 3b = 11.25

10c * 3 = 30c

3b * 3 = 9b

11.25 * 3 = 33.75

30c + 9b = 33.75

Now we can subtract the second equation from this:

30c - 6c = 24c

9b - 9b = 0

33.75 - 15.75 = 18

24c = 18

Now divide both sides by c to find 1c:

c = 0.75

Now put this back into the first equation:

10(0.75) + 3b = 11.25

7.50 + 3b = 11.25

Move the +7.50 over to the other side making it a negative:

3b = 11.25 - 7.50

3b = 3.75

Divide both sides by 3:

b = 1.25

So one brownie costs $1.25
Hope this helps!

Write an expression to represent: Three more than the product of two and a number x.

Answers

Answer:

2x + 3

Step-by-step explanation:

product means to multiply, hence

the product of 2 and a number x = 2 × x = 2x

3 more than means add on 3, thus

expression is 2x + 3


If U = {natural numbers less than 20} and N = {factors of 18}, what is N

Answers

U={0,1,2,3, 4,5,6,7,8,9,10,11,12,13,14,15,16,17,18,19}
N={1,2,3,6,9,18}

This is from the answer key to this question;

N= {4, 5, 7, 8, 10, 11, 12, 13, 14, 15, 16, 17, 19.}

Solve 14.2 divided by 0.5 dhow your work and explain how you new where thr decamal point whas going to be

Answers

Final answer:

To solve the division problem 14.2 divided by 0.5, you can use the long division method. The quotient is 28.4.

Explanation:

To solve the division problem 14.2 divided by 0.5, we can perform the long division method. Here's the step-by-step process:

Place the dividend, 14.2, inside the division bar.Place the divisor, 0.5, outside the division bar.Divide the first digit of the dividend, which is 1, by the divisor. The result is 2.Write down the quotient, 2, above the division bar.Multiply the quotient, 2, by the divisor, 0.5, which gives you 1.Subtract the product, 1, from the first digit of the dividend, 1, which gives you a remainder of 0.Bring down the next digit, which is 4, from the dividend.Divide the new dividend, 4, by the divisor, 0.5. The result is 8.Write down the new quotient, 8, next to the previous quotient.Multiply the new quotient, 8, by the divisor, 0.5, which gives you 4.Subtract the product, 4, from the new dividend, 4, which gives you a remainder of 0.

Since there is no remainder anymore, the division is complete. The final quotient is 28.4. Therefore, 14.2 divided by 0.5 is equal to 28.4.

The slope of the line below is-1/7Write a point-slope equation of the line using the coordinates of the labeled point.

Answers

Answer:

While you do not have the coordinates included, you can use the point-slope form of the equation and input the slope. This would be:

y - y1 = -1/7(x - x1)

In this equation, simply put the ordered pair in at x1 and y1. This will give you the full point-slope equation.


Hat is the position of 7 in the number 876,543? A. The ten-thousands place B. The hundreds place C. The tens place D. The thousands place

Answers

As you can see in this diagram, the answer is A, the ten-thousands place.

Someone please help with these questions

Answers

Question 4:

  Statement                                           Reason

1. [tex]\overline{AC}[/tex] and [tex]\overline{BD}[/tex] bisect at O                    1. Given

2. O is the midpoint of [tex]\overline{AC}[/tex] and [tex]\overline{BD}[/tex]   2. Definition of bisector

3. [tex]\overline{AO}[/tex] ≅ [tex]\overline{CO}[/tex] and [tex]\overline{BO}[/tex]  ≅ [tex]\overline{DO}[/tex]                3. Definition of midpoint

4. ∠AOB ≅ ∠COD                                4. Vertical Angle theorem

5. ΔAOB ≅ ΔCOD                                5. SAS postulate

6. ∠A ≅ ∠C                                           6. CPCTC

*****************************************************************************

Question 5:

  Statement                                    Reason

1.  [tex]\overline{BD}[/tex] ≅ [tex]\overline{BC}[/tex] and [tex]\overline{AD}[/tex] ≅ [tex]\overline{CD}[/tex]         1. Given

2. [tex]\overline{BD}[/tex] ≅ [tex]\overline{BD}[/tex]                                 2. Reflexive Property

3. ΔABD ≅ ΔCBD                         3. SSS Postulate

4. ∠1 ≅ ∠2 and ∠3 ≅ ∠4               4. CPCTC

5. [tex]\overline{BD}[/tex] bisects ∠ABC and ∠ADC  5. Definition of angle bisector

HELP!!!!!! I believe its B but I am not sure! :(

Answers

Answer:

A

Step-by-step explanation:

When solving for x as an exponent, we need to use logarithms in order to undo the operation and rearrange the terms. We use log rules to bring down the exponent and solve. Logarithms are the inverse operations to exponents and vice versa. We have one special kind of logarithm called the natural logarithm whose base is e. We write it as ln. Since our base is e here, we will use the natural logarithm to rearrange and isolate x.

[tex]e^{4x-1} =3[/tex]

We begin by applying the natural logarithm to each side.

[tex]ln(e^{4x-1}) =ln(3)[/tex]

Log rules allow use to rearrange the exponent as multiplication in front of the log.

[tex](4x-1)ln(e) =ln(3)[/tex]

ln e as an inverse simplifies to 1.

[tex](4x-1)(1)=ln(3)[/tex]

We now apply the inverse operations for subtraction and multiplication.

[tex]4x-1+1=1+ln(3)\\4x=1+ln(3)\\\frac{4x}{4} =\frac{1+ln3}{4} \\x =\frac{1+ln3}{4}[/tex]

Option A is correct.

Kris has a box of 8 crayons. Silvia, s box has 6 times of many crayons as Kris box . How many crayons are in sylvia box

Answers

Sylvia would have 48 crayons.
6•8=48

Ron buys pens from a store. 3 packs of blue pens cost $22.50 and 4 packs of black pens cost $34.00. How much would it cost Ron to buy 2 packs of blue pens and 3 packs of black pens?

Answers

The blue pens would cost $15 and the black ones would cost $25.50. In total that is $40.50.

Use mapping notation to represent a 90 degree clockwise rotation about the origin of the point (x, y)

Answers

Answer:

see explanation

Step-by-step explanation:

Under a clockwise rotation about the origin of 90°

a point (x , y) → (y , - x)


Answer:The answer is choice C. and the rotation is 180

Step-by-step explanation:i just got it right

Find the slope-intercept form of the equation 3x + 7y = 14.

A. 7y = 3x + 14

B. 7y = -3x + 14

C. y= (-3)/7 x+2

D. y= 3/7 x+2

Answers

The slope-intercept form:

[tex]y=mx+b[/tex]

We have:

[tex]3x+7y=14[/tex]              subtract 3x from both sides

[tex]7y=-3x+14[/tex]             divide both sides by 7

[tex]\boxed{y=-\dfrac{3}{7}x+2\to\boxed{C.}}[/tex]

What is the Domain?


What is the Range?


Answers

[tex]f(x)=5x\\\\\text{The domain:}\ D=\{0,\ 1,\ 2,\ 3,\ 4,\ 5,\ 6\, ...\}=\text{Whole numbers}\\\\\text{The range:}\\\\f(0)=5(0)=0\\f(1)=5(1)=5\\f(2)=5(2)=10\\f(3)=5(3)=15\\f(4)=5(4)=20\\f(5)=5(5)=25\\\vdots\\R=\{0,\ 5,\ 10,\ 15,\ 20,\ 25,\ ...\}[/tex]

Triangle ABD is congruent to Triangle DFE. If the length of line segment BC is 5 units, what is the length of line segment EF?

Answers

Answer:

EF = 5 units

Step-by-step explanation:

If ΔABC and ΔDFE are congruent, then

AB = DF

BC = FE

CA = ED

and

angles A and D are congruent

angles B and F are congruent

angles C and E are congruent


1 yard = 3 feet 1 foot = 12 inches 1 inch = 2.54 cm Convert 2 yards to centimeters (round to the nearest whole centimeter).

Answers

The measure of 2 yards in centimeters is 182.88 cm

Using the conversions given:

1 inch = 2.54 cm

1 yard = 3 × 12 inches

1 yard = 36 inches

1 yard = 36 × 2.54 cm

1 yard = 91.44 cm

Hence,

For 2 yards : (91.44 × 2) = 182.88 cm

The measure of 2 yards in centimeters is 182.88 cm

There are 64 pretzels in a 16-ounce bag of chocolate covered pretezel. What is the rate of pretzels per ounce?

Answers

the answer is 4 pretzels

Ms.Rochelle wants to put her 29 students into groups of 6. How many groups of 6 can she make? If she puts any remaining students in a smaller group, how many students will be in that group?

Answers

Answer:

A. 4 groups of 6 students in each group.

B. The smaller group will have 5 students.        

Step-by-step explanation:  

We have been given that Ms.Rochelle wants to put her 29 students into groups of 6.  

A. To find the number of groups of 6 that can be made from 29 students, we will find the greatest multiple of 6 from 29.    

Multiples of 6 are: 6, 12, 18, 24, 30, 36,...  

We can see that the greatest multiples of 6 in 29 in 24.  

[tex]\text{Number of groups that can be made with 29 students}=\frac{24}{6}[/tex]          

[tex]\text{Number of groups that can be made with 29 students}=4[/tex]

Therefore, Ms. Rochelle can form 4 groups from 29 students with 6 students in each group.            

B. To find the number of students in small group, we will subtract 24 from 29.  

[tex]\text{Number of students in the small group}=29-24[/tex]

[tex]\text{Number of students in the small group}=5[/tex]  

Therefore, If Ms. Rochelle puts any remaining students in a smaller group, the number of students in smaller group will be 5.  


x+2= √2 solve for x will give brainliest

Answers

Answer:

Our answer is [tex]x=-2+\sqrt{2}[/tex].

Step-by-step explanation:

To solve for x, we need to isolate the variable. We can do so by subtracting 2 to both sides.

[tex]x+2=\sqrt{2} \\x+2-2=-2+\sqrt{2} \\\\x=-2+\sqrt{2} \\[/tex]

Our answer is [tex]x=-2+\sqrt{2}[/tex].

To find the decimal, calculate with a calculator.

Jenny makes and sells bracelets. She paid $12.75 for the materials to make 45 bracelets. If she sells every bracelet for $1.50 each, how much profit will she earn?

Answers

Answer is in the picture attached.
Final answer:

Jenny's profit per bracelet is about $1.22. If she sells all 45 bracelets, her total profit is approximately $54.77.

Explanation:

The subject of this question is profit, which in this case will occur when Jenny is able to sell her bracelets for more than she paid for the materials to make them. She pays $12.75 for the materials to make 45 bracelets, meaning each bracelet costs her $12.75 / 45 = $0.283 to make. If she sells each bracelet for $1.50, that means she earns $1.50 - $0.283 = $1.217 in profit for each bracelet. If she sells all 45 bracelets, her total profit would therefore be $1.217 * 45 = $54.765. However, since we typically round money to the nearest cent, her total profit will be approximately $54.77.

Learn more about Profit Calculation here:

https://brainly.com/question/32944523

#SPJ2

Other Questions
Which absolute value function, when graphed, represents the parent function, f(x) = |x|, reflected over the x-axis an Which absolute value function, when graphed, represents the parent function, f(x) = |x|, reflected over the x-axis and translated 1 unit to the right?d translated 1 unit to the right? the energy role of a grizzly bear is that of a(n) ________ because it cannot make it's own food what is termed as virgin vegetation ? how much natural is the natural vegetation of india today ? 8 out of the 20 students at a school assembly were first-grade students. What percentage of the students at the assembly were first-graders? Write your answer using a percent sign (%) Which of the following is an example of acceleration? Select one: a. A boy walks two miles in one hour. b. A girl walks two miles per hour north for three hours. c. A boy walks two meters north, turns west, and walks another two meters. d. A girl drives a total distance of 240 miles in just under four hours. 7.A modern, popular way to deliver story used in franchise like Halo and half life is:A. to begin with uncertainty and resolved over the course of timeB. too ra play over and over each time a player fails a challengeC. to make large and complex Stories part of games even when it's not a focal pointD. to deliver story via interactions with AI characters so that it's not directly connected to game actions8. a game that makes his puzzles or problem-solving within Bayern mental action and dialogue with characters would most likely be a(n)_______ game.A. role-playing B. survival C. puzzle D. adventure At 55 mph it will take you approximately _____ feet to stop the car. A. 5 feet B. 100 feet C. 228 feet Evaluate 3x 2 if x = 6.1620186 How do I find the distance from the epicenter? Please show me step by step of how to find the distance from the epicenter. See attached picture. PLEASE HELP!!!Sam (4, 2) Theater (-3, 5) Describe Sam's location relative to the theater. Which statement about folk art landscapes is true? Which is the graph of the equation y2=0.5(x+3)? Dr. macpherson believes that the way students organize and think about the information in their textbooks will strongly influence their ability to later remember and use what they have studied. dr. macpherson's ideas most directly exemplify the _____ perspective. behaviorist name 2 essential roles that enzymes play in cells How to get 5% of 220 files A.) A diver descends 12 feet below sea level. She then ascends 4 feet. State the depth of the diver in terms of sea level. Explain how the depth was determined. B.) Suppose the diver now descends an additional 12 feet. State the new depth of the diver in terms of sea level. Explain how this depth was determined. C. ) Explain how the diver will reach sea level from the position found in part B. 22-228-8-2020-44Swim up 4 feetSwim up 20 feetSwim up 8 feetSwim up 20 feet Answer for part A: Answer for part B: Answer to part C: Which list shows the levels of organization of an organism in hierarchical order from left to right, from the smallest to the most complex? A organism, organ, organ system, cell, B tissue organism, organ system, organ, tissue, cell C cell, tissue, organ system, organ, organism D cell, tissue, organ, organ system, organism Factor x^3-4x^2-3x+18=0 given that 3 is a zero.A) (x+2)(x-3)^2=0B) (x-2)(x-3)^2=0C) (x-2)(x-3)(x+3)=0D) (x+2)(x-\sqrt{3})(x+\sqrt{3})=0 n triangle ABC, mBAC = 50. If mACB = 30, then the triangle is triangle. If mABC = 40, then the triangle is triangle. If triangle ABC is isosceles, and AB = 6 and BC = 4, then AC = Escribe la forma correcta del verbo para completar la frase. Write the correct form of the verb to complete the sentence. You may copy and paste the accented and special characters from the following list if needed: , , , , , , , , , , , , , , Cuando yo era nia, yo ________(vivir) en Argentina. Steam Workshop Downloader